0% found this document useful (0 votes)
157 views

MATH5011 Solution 3 2017 Medida

This document contains solutions to exercises from a real analysis course. The solutions include: 1) Proving that Lebesgue's dominated convergence theorem still holds if the condition of almost everywhere convergence is replaced with convergence in measure. 2) Providing examples of sequences that converge in measure but not everywhere, converge everywhere but not in measure, and converge in measure but not in L1. 3) Showing that a sequence converges in measure if and only if every subsequence has a sub-subsequence that converges almost everywhere.

Uploaded by

Omonda Nii
Copyright
© © All Rights Reserved
We take content rights seriously. If you suspect this is your content, claim it here.
Available Formats
Download as PDF, TXT or read online on Scribd
0% found this document useful (0 votes)
157 views

MATH5011 Solution 3 2017 Medida

This document contains solutions to exercises from a real analysis course. The solutions include: 1) Proving that Lebesgue's dominated convergence theorem still holds if the condition of almost everywhere convergence is replaced with convergence in measure. 2) Providing examples of sequences that converge in measure but not everywhere, converge everywhere but not in measure, and converge in measure but not in L1. 3) Showing that a sequence converges in measure if and only if every subsequence has a sub-subsequence that converges almost everywhere.

Uploaded by

Omonda Nii
Copyright
© © All Rights Reserved
We take content rights seriously. If you suspect this is your content, claim it here.
Available Formats
Download as PDF, TXT or read online on Scribd
You are on page 1/ 10

2017 Fall Real Analysis I 1

Suggested Solutions to Exercise 3

Standard notations are in force.

(1) Prove the conclusion of Lebsegue’s dominated convergence theorem still holds
when the condition “{fk } converges to f a.e.” is replaced by the condition “
{fk } converges to f in measure”.
Z
Solution. Suppose on the contrary that |fk − f |dµ does not tend to zero.
By considering the limit supremum of the sequence, we can find a positive
constant M and a subsequence {fnj } such that
Z
|fnj − f |dµ ≥ M

for all j. By Prop 1.17, a subsequence {gk } of {fnj } converges to f a.e.. By


Lebsegue’s dominated convergence theorem, we have
Z
0 = lim |gk − f |dµ ≥ M > 0,
k→∞

contradiction holds.

(2) Find an example in each of the following cases.

(a) A sequence which converges in measure but not at every point.

(b) A sequence which converges pointwisely but not in measure.

(c) A sequence which converges in measure but not in L1 .

Solution.

(a) Take X = [0, 1] and the Lebesgue measure. Basically, let f1 = χ[0,1/2] , f2 =
χ[1/2,1] , f3 = χ[0,1/22 ] , f4 = χ[1/22 ,1/2] , f5 = χ[1/2,3/22 ] , f6 = χ[3/22 ,1] , · · · .
2017 Fall Real Analysis I 2

(b) According to Proposition 1.18, the measure cannot be finite. Take X =


R and the Lebesgue measure. Consider gn = χ[n,n+1] . We have gn (x) →
0 for all x but not in measure.

(c) Take X = R and the Lebesgue measure. Consider hn = n2 χ[0,1/n] . Then


hn → 0 in measure but not in L1 .

(3) Let fn , n ≥ 1, and f be real-valued measurable functions in a finite mea-


sure space. Show that {fn } converges to f in measure if and only if each
subsequence of {fn } has a subsubsequence that converges to f a.e..

Solution. Let µ be the measure in a finite measure space. If {fn } converges


to f in measure, then every subsequence {fnk } also converges to f in measure.
By Prop 1.17, the subsequence {fnk } has a sub-subsequence converging to f
a.e..

Now suppose that each subsequence of {fn } has a sub-subsequence that con-
verges to f a.e.. Assume that fn does not converge to f in measure. By
considering the limit supremum, there are positive ρ , M and subsequence
{fnj } such that, µ({x : |fnj (x) − f (x)| ≥ ρ}) ≥ M , for all j. However, a sub-
sequence {gk } of {fnj } converges to f a.e. and by Prop 1.18, {gk } converges
to f in measure and

0 = lim µ({x : |gk (x) − f (x)| ≥ ρ}) ≥ M > 0,


k→∞

which is impossible. Hence {fn } converges to f in measure.

(4) Let (X, M, µ) be a measure space. Let M


f contain all sets E such that there

exist A, B ∈ M, A ⊂ E ⊂ B, µ(B\A) = 0. Show that M


f is a σ-algebra

containing M and if we set µ


e(E) = µ(A), then (X, M,
f µe) is a complete
measure space.

Solution. We see that M


f contains M by taking E = A = B for any E ∈

M. Suppose Ei ∈ M,
f Bi ⊆ Ei ⊆ Ai where Bi , Ai ∈ M and µ(Ai \Bi ) = 0,
2017 Fall Real Analysis I 3

then

\ ∞
\ ∞
\
Bi ⊆ Ei ⊆ Ai
i=1 i=1 i=1

and

\ ∞
\ ∞
[ ∞
X
µ( Ai \ Bi ) ≤ µ( Ai \Bi ) ≤ µ(Ai \Bi ) = 0.
i=1 i=1 i=1 i=1


\
We have Ei is in M.
f If A ⊇ E ⊇ B, then
i=1

X\A ⊆ X\E ⊆ X\B

and
µ((X\B)\(X\A)) = µ(A\B).

Hence X\E is in M
f and M
f is a σ algebra. We check that µ
e is a measure
on M
f. Obviously µ
e(φ) = 0. Let Ei be mutually disjoint µ
e measurable set,
∃Bi , Ai µ measurable s.t
Ai ⊆ Ei ⊆ Bi

and
µ(Bi \ Ai ) = 0.


[ ∞
[
Using above argument, we have µ( Bi \ Ai ) = 0, And Ai are mutually
i=1 i=1
disjoint,

[ ∞
[ ∞
X ∞
X
µ
e( Ei ) = µ( Ai ) = µ(Ai ) = µ
e(Ei ).
i=1 i=1 i=1 i=1

So µ
e is a measure on M
f.

Finally, we check that µ


e is a complete measure, let E be a µ
e measurable and
null set, for all subset C ⊆ E, we have ∃A, B ∈ M s.t.A ⊆ E ⊆ B and
2017 Fall Real Analysis I 4

µ(A) = µ(B) = 0. Therefore

φ⊆C⊆B

and
µ(B) = 0.

We have C ∈ M.
f

(5) Show that M


f in the previous problem is the σ-algebra generated by M and

all subsets of measure zero sets in M.

Solution. Let M1 be the σ-algebra generated by M and all subsets of


measure zeros sets in M.

By definition, M
f contains all the sets in M and all subsets of measure zero

sets in M. Since M1 is the smallest such σ-algebra, we have M1 ⊂ M.

To prove that M
f ⊂ N , we let E ∈ M.
f Then there exist A, B ∈ M such that

A ⊂ E ⊂ B and µ(B \ A) = 0. Then E \ A ⊂ B \ A is a subset of a measure


zero set. Now E = A ∪ (E \ A) is a union of a set in M and a subset of a
measure zero set. Hence, E ∈ M1 .

(6) Here we consider an application of Caratheodory’s construction. An algebra


A on a set X is a subset of PX that contains the empty set and is closed under
taking complement and finite union. A premeasure µ : A → [0, ∞] is a finitely
additive function which satisfies: µ(φ) = 0 and µ( ∞
S P∞
k=1 Ek ) = k=1 µ(Ek )
S∞
whenever Ek are disjoint and k=1 Ek ∈ A. Show that the premeasure µ can
be extended to a measure on the σ-algebra generated by A. Hint: Define the
outer measure

X [
µ(E) = inf µ(Ek ) : E ⊂ Ek , Ek ∈ A .
k k
2017 Fall Real Analysis I 5

This is called Hahn-Kolmogorov theorem.

Solution. We follow the proof from Terence Tao’s book “Introduction To


Measure Theory”. Define µ as above and obviously µ is an outer measure on
power set of X. By Caratheodory’s construction, we get a measure defined
on a σ- algebra M . We claim that A ⊆ M , let E ∈ A and C ⊆ X such that
µ(C) < ∞, for all ε > 0, there is {Ei }∞
i=1 ⊆ A covering C such that


X
µ(C) + ε > µ(Ei ).
i=1

As {Ei ∩ E}∞ ∞
i=1 and {Ei \ E}i=1 are subset of A and cover C ∩ E and C \ E

respectively, we have,


X
µ(C ∩ E) ≤ µ(Ei ∩ E),
i=1

and

X
µ(C \ E) ≤ µ(Ei \ E).
i=1

Using the fact that µ is a premeasure, µ(Ei ∩ E) + µ(Ei \ E) = µ(Ei ). Sum-


ming over i, we know that E is in M and M contains the σ algebra generated
by A. Now we try to show that the measure induced extends µ, obviously
by definition µ(E) ≤ µ(E). Let {Ei }∞
i=1 ⊆ A covering E. Without affect-

ing the countable union, we may make {Ei }∞ ∞


i=1 disjoint and obtain {Bi }i=1 .

Furthermore, by taking intersection with E, we have


[
Bi ∩ E = E
i=1

and

X ∞
X ∞
X
µ(Ei ) ≥ µ(Bi ) ≥ µ(Bi ∩ E) = µ(E),
i=1 i=1 i=1

where the last equality follows from the condition of µ. Hence µ(E) ≥ µ(E)
2017 Fall Real Analysis I 6

and the measure extends µ.

The following problems are concerned with the Lebesgue measure. Let R =
I1 ×I2 ×· · ·×In , Ij bounded intervals (open, closed or neither), be a rectangle
in Rn . More properties of the Lebesgue measure can be found in Exercise 4.

(7) For a rectangle R in Rn , define its “volume” to be

|R| = (b1 − a1 ) × (b2 − a2 ) × · · · × (bn − an )

where bi , ai are the right and left endpoints of Ij . Show that


N
[
(a) if R = Rk where Rk are almost disjoint, then
k=1

N
X
|R| = |Rk |.
k=1

N
[
(b) If R ⊂ Rk , then
k=1
N
X
|R| ≤ |Rk |.
k=1

Solution.

(a) Take n = 2 for simplicity. Each rectangle is of the form [a, b] × [c, d]. We
order the endpoints of the x-coordinates of all rectangles R1 , · · · , RN into
a1 < a2 < · · · < an and y-coordinates into b1 < b2 < · · · < bm . This division
breaks R into an almost disjoint union of subrectangles Rj,k = [aj , aj+1 ] ×
[bk , bk+1 ]. Note that each Rj,k is contained in exactly one Rl and each Rl is
2017 Fall Real Analysis I 7

an almost disjoint union of subrectangles from this division. We have

|R| = (an − a1 )(bm − b1 )

= (an − an−1 + an−1 − an−2 + · · · + a2 − a1 )(bm − bm−1 + bm−1 + · · · + b2 − b1 )


X
= |Rj,k |
j,k
X X
= |Rj,k |
l Rj,k ⊆Rl
X
= |Rl |,
l

(b) The proof is similar to that of (a), but now we need to subdivide Rj
N
[
and R together. Now, R ⊆ Rj . We order all x-coordinates of Rj , R into
j=1
a1 < a2 < · · · < aN and and y-coordinates into b1 < b2 < · · · < bM . Then R
is the union of parts of Rk,j

X
|R| = |Rj,k |
Rj,k ⊆R
X
≤ |Rj,k |
j,k
X
≤ |Rj |,
j

where the last inequality follows from the fact that each Rk,j is contained in
some Rj .

(8) Let R be the collection of all closed cubes in Rn . A closed cube is of the form
I × · · · × I where I is a closed, bounded interval.

(a) Show that (R, | · |) forms a gauge, and thus it determines a complete
measure Ln on Rn called the Lebesgue measure.

(b) Ln (R) = |R| where R is a cube, closed or open.


2017 Fall Real Analysis I 8

(c) For any set E and x ∈ Rn , Ln (E + x) = Ln (E). Thus the Lebsegue


measure is translational invariant.

Solution.

(a) We clearly have

[
inf |R| = 0 and R = Rn .
R∈R
R∈R

Hence, R forms a gauge.

Define µ by (∞ )
X
µ(A) = inf |Gj | : Gj ∈ R .
j=1

We check that µ is an outer measure.

• Clearly, µ(φ) = 0.
S
• Suppose {Ej : j ∈ N } are given and write E = j∈N Ej . Let ε > 0.
Choose Gjk ∈ R such that

X
µ(Ej ) + 2−j ε > |Gjk |.
k∈N

Then {Gjk : j, k ∈ N} is a countable cover for E. We have

X X X
µ(E) ≤ |Gjk | < µ(Ej ) + 2−j ε = µ(Ej ) + ε.
j,k∈N j∈N j∈N

Taking ε → 0, we have

X
µ(E) ≤ µ(Ej ).
j∈N

Following the Carathéodory’s construction, we obtain a complete mea-


sure.
2017 Fall Real Analysis I 9


[ ∞
X
(b) By (9)(b), it suffices to show that R ⊆ Rj ⇒ |R| ≤ |Rj |. We
replace Rj = [aj , bj ] × [cj , dj ] by Ŕj = (aj − 2εj , bj + 2εj ) × (cj − 2ε , dj + 2ε ).
Since {Ŕj } is an open cover of R and R is compact, there exists a finite
subcover R´j1 , · · · , R´jM . By (9)(b)

M
X X X
|R| ≤ |R´jk | ≤ |Rj,k | ≤ |Rj | + Cε,
k=1 j,k j

C depends on n only. Let ε → 0,

X
|R| ≤ |Rj |,
j

which shows that


X ∞
[
|R| = inf{ |Rj | : R ⊆ Rj , Rj closed cube}.
1 j=1

Therefore,
Ln (R) = |R|

where R is a closed cube.


In order to show it also holds for any cube, it suffices to show Ln (F ) = 0
whenever F is a face of R. First, Ln (F ) ≤ Ln (R) < ∞. Let N be
any number> 1 and x1 = (a1 , 0), · · · , xN = (aN , 0) be distinct point.
Consider F + xj . For small aj , F + xj can be chosen to sit inside R, then
[ [
(F + xj ) ⊂ R. By (c), N Ln (F ) = Ln (F + xj ) = Ln ( (F + xj )) ≤
P

n n Ln (R)
L (R) ⇒ L (F ) ≤ → 0 as N → ∞.
N
(c) Result follows directly from definition.
2017 Fall Real Analysis I 10

(9) This problem is optional. Use Hahn-Kolmogorov theorem to construct the


Lebesgue measure instead of Problems 7 and 8.

Solution. Look up Wiki.

You might also like